what is the volume of the can in cubic inches in terms of

What Is The Volume Of The Can In Cubic Inches In Terms Of

Answers

Answer 1

Given data:

The height of the cylinder is h=9 in.

Th diameter of the cylinder is d=6 in.

The expression for the volume of the cylinder is,

V=(πd^2h)/4

Substitute the given values in the above expression.

[tex]\begin{gathered} V=\pi(6in)^2(9\text{ in)}\frac{1}{4} \\ =81\pi in^3 \end{gathered}[/tex]

Thus, the volume of the given figure is 81 in^3. so C) option is correct.


Related Questions

Find the derivatives of the following using the different rules.1. y = 3x + 29

Answers

To find the derivative of the given function, we can use the power rule.

[tex]ax^n\Rightarrow nax^{n-1}[/tex]

In this rule, we multiply the exponent of the variable by its numerical coefficient and then subtract 1 from the exponent.

For this function y = 3x + 29, we have two terms. These are 3x and 29. We need to apply the power rule for each term.

Let's start with 3x.

[tex]3x^1\Rightarrow1(3)(x^{1-1})\Rightarrow3x^0\Rightarrow3[/tex]

The first derivative for 3x is 3.

For the term 29, since there is no variable, the derivative for 29 is 0.

So, the first derivative of y = 3x + 29 is y' = 3 + 0 or just y' = 3.

[tex]y^{\prime}=3[/tex]

Select the correct choice below and fill in any answer boxes in your choice.

Answers

Answer;

[tex]x=40[/tex]

Explanation;

To get the correct choice, we have to simplify the given equation as follows;

[tex]\begin{gathered} 4x-(2x+6)=3x-46 \\ 4x-2x-6=3x-46 \\ 2x-6=3x-46 \\ 3x-2x=46-6 \\ x\text{ = 40} \end{gathered}[/tex]

So, the correct option is A and we fill the value 40 in the box

Can someone please help me with these, please? I’ve tried them myself already, but I got confused enough I didn’t end up with an answer

Answers

Given:

The cost for a day food, entertainment and hotes is $250.

The cost for round trip air fair is $198.

Explanation:

Let x represents the number of full days that individual can stay at the beach.

The total money available to individual is $1400. So inequality is,

[tex]\begin{gathered} 250\cdot x+198\leq1400 \\ 250x+198\leq1400 \end{gathered}[/tex]

Thus inequality for number of days is,

[tex]250x+198\leq1400[/tex]

The variable x represent the number of full days that individual can spend at beach trip.

(b)

Solve the inequality for x.

[tex]\begin{gathered} 250x+198-198\leq1400-198 \\ \frac{250x}{250}\leq\frac{1202}{250} \\ x\leq4.808 \end{gathered}[/tex]

The maximum whole value of x is 4.

Thus individual can spend 4 complete (full) days at the beach trip.

Divide the polynomials using long division or synthetic division.
(x³ − 4x²+x+6) ÷ (x − 2)
(Type your answer in the box WITHOUT any spaces.)
I
A/

Answers

Answer:To divide a polynomial by a binomial of the form  x - c using synthetic division.

Use the Remainder Theorem in conjunction with synthetic division to find a functional value.

Use the Factor Theorem in conjunction with synthetic division to find factors and zeros of a polynomial function.

Step-by-step explanation:

if you have the lessons this is what id do to solve this!

can you help me with this question

Answers

Since B is the midpoint of AC, we can conclude:

[tex]\begin{gathered} AC=AB+BC \\ also \\ AB=BC \\ so\colon \\ 6x+1=2x+9 \\ solve_{\text{ }}for_{\text{ }}x\colon \\ 6x-2x=9-1 \\ 4x=8 \\ x=\frac{8}{4} \\ x=2 \end{gathered}[/tex]

A TV set is offered for sale for P1, 800 down payment, and P950 every month for thebalance for 2 years. If interest is to be computed at 6% compounded monthly, what isthe cash price equivalent of the TV set?

Answers

Answer:

The cash equivalent price of the tv set is P5849.37

Explanation:

The total amount paid monthly is:

The initial down payment of P1800 and 24 payments of P950 (2 years = 24 months)

Then, the price paid with interest is:

[tex]1800+24\cdot950=24600[/tex]

Now, the formula for the monthly compound interest is:

[tex]A=P(1+\frac{r}{12})^{12t}[/tex]

Where:

A is the amount after t years

P is the initial amount (what we want to find)

R is the rate of compound in decimal

t is the number of years of compounding

In this case:

• A = 24600 (the total paid if compounded)

,

• P is what we want to find

,

• r = 0.06 ( to convert percentage to decimal, we divide by 100: 6%/100 = 0.06)

,

• t = 24 (2 years)

[tex]\begin{gathered} 24600=P(1+\frac{0.06}{12})^{12\cdot24} \\ . \\ 24600=P\cdot1.005^{288} \\ . \\ P=\frac{24600}{4.2056} \\ . \\ P=5849.373 \end{gathered}[/tex]

To the nearest cent, the cash price is 5849.37

Can you please explain how to differentiate an equation? specifically, how to get from this:h(t) = -16t^2 + 72t + 24 to this:h'(t) = -32t + 72I am a parent trying to help my child. looks vaguely familiar but it's been a long time, if you know what I mean! Thank you!

Answers

[tex]h(t)=-16t^2+72t+24[/tex]

To differentiate an equation as given you can use the next:

Derivates of powers:

[tex]\begin{gathered} f(x)=x^n \\ f^{\prime}(x)=nx^{n-1} \\ \\ \\ f\mleft(x\mright)=x \\ f^{\prime}(x)=1 \\ \end{gathered}[/tex]

Derivate of a constant:

[tex]\begin{gathered} f(x)=c \\ f^{\prime}(x)=0 \end{gathered}[/tex]

You have in the given equation two powers (the fist two terms) and a constant (las term (24)):

[tex]\begin{gathered} h^{\prime}(t)=-2(16t)^{2-1}+72(1)+0 \\ \\ h^{\prime}(t)=-32t+72 \end{gathered}[/tex]

May I please get help with finding out weather each of them can be the HL congruence property

Answers

The hypotenuse-leg theorem states that two right right triangles are congruent if the hypotenuse and a leg of one triangle are congruent to the hypotenuse and a leg of the other triangle. Looking at the given options,

For a,

We only know that two legs are congruent. We can't confirm that the hypotenuses are congruent

For b,

two legs and two hypotenuses are congruent

For c, the triangles don't have hypotenuses because they are not right triangles.

For d, the hypotenuses of both triangles is the common line. This means that they are congruent. Two legs are also congruent.

Thus, the correct options are

b. Yes

d. Yes

Mark says that an angle showing a turn through of a circle that is 10 inches across

Answers

The question involves comparing the measure of two angles different circles

For the first circle

The angle turn through 1/4 of a circle, which means that the circle turns through 90 degrees

For the second circle that is 10 inches across

The angle also turned through 1/4 of the circle. This also implies that the circle turns through 90 degrees

Thus,

The angle turned by the two circles is the same. Hence, Mark is not correct

2 + 2 what is it I need to know please :/

Answers

2+2=4

The answer is 4

Answer:

4!

Step-by-step explanation:

when you add them together you get 4

In the Itty Bitty High School, there are 85 students. There are 27 students whotake French, 51 who take Geometry and 38 who take History. There are 10 thattake Geometry and French, 7 that take History and French and 15 that takeGeometry and History. There are 2 students who take all 3 and 5 that take noneof these subjects.What it the probability that you randomly select a person who is a female giventhey are brown headed? *Fair ColorBrom Blonde Red87682SaleFemale34861271666Your answer

Answers

ANSWER:

2.2%

STEP-BY-STEP EXPLANATION:

The probability would be the quotient between the number of people with these characteristics (female, brown hair) and the total number of people

[tex]\begin{gathered} p(\text{female and brown) }=\frac{66}{548+876+82+612+716+66}=\frac{66}{1450}=0.022 \\ p(\text{female and brown) }=2.2\text{\%} \end{gathered}[/tex]

Factor the given polynomial completely and match your result to the correct answer below.18m³ +24m²-24mSelect one:O a. 6m(m-4)(3m + 1)O b. 6m(3m2 +6m-4)O c.6m(m+2)(3m-2)O d. The polynomial is prime.

Answers

Given:

[tex]18m^^3+24m^2-24m[/tex]

Required:

We need to factorize the given polynomial completely.

Explanation:

Take out the common multiple 6m.

[tex]18m^3+24m^2-24m=6m(3m^2+4m-4)[/tex][tex]Use\text{ 4m=6m-2m.}[/tex]

[tex]18m^3+24m^2-24m=6m(3m^2+6m-2m-4)[/tex]

Take out the common multiple.

[tex]18m^3+24m^2-24m=6m(3m(m+2)-2(m+2))[/tex][tex]18m^3+24m^2-24m=6m(m+2)(3m-2)[/tex]

Final answer:

[tex]6m(m+2)(3m-2)[/tex]

If the ratio of AB to BC is 11:6, at what fraction of AC is point B located? Round to the nearesthundredth, if necessary.

Answers

For this case we know that the ratio of AB to BC is 11:6 and we can set up the following ratio:

[tex]\frac{AB}{AC}=\frac{11}{6}[/tex]

And we want to identify what fraction of AC is point B located

We can assume that the lenght of AC is lower than AB

So then we can answer this problem with this operation:

[tex]\frac{6}{11}=0.545[/tex]

And the answer for this case would be 0.545

Instead, now suppose that P(x) = 5band b = 2. What is the weekly percent growth rate in thiscase? What does this mean in every-day language?

Answers

According to the given value of b, we can determine the growth rate by substracting 1 from the value of b, that is 2, and converting the result to a percent:

[tex]\begin{gathered} 2-1=1 \\ 1\cdot100=100\% \end{gathered}[/tex]

It means that the weekly growth rate is 100%.

In every day language, it means that every week, the number of fish in the lake doubles the number of the last week.

the speed of a stream is 4 mph. A boat travels 6 miles upstream in the same time it takes to travel 14 miles downstream. what is the speed of the boat in still water?

Answers

The speed of the boat in still water is 10 mph.

What is a expression? What is a mathematical equation?

A mathematical expression is made up of terms (constants and variables) separated by mathematical operators. A mathematical equation is used to equate two expressions. Equation modelling is the process of writing a mathematical verbal expression in the form of a mathematical expression for correct analysis, observations and results of the given problem.

We have the speed of a stream as 4 mph. A boat travels 6 miles upstream in the same time it takes to travel 14 miles downstream.

Assume the speed of the boat in still water be [x] mph.

We know that -

time [t] = distance[x]/speed[s]

Then, we can write -

6/(x - 4) = 14/(x + 4)

6(x + 4) = 14(x - 4)

6x + 24 = 14x - 56

56 + 24 = 8x

80 = 8x

x = 10 mph

Therefore, the speed of the boat in still water is 10 mph.

To solve more questions on Equations, Equation Modelling and Expressions visit the link below -

brainly.com/question/14441381

#SPJ1

2. What is the value of 'm' if 9m ÷ 9-3 = 95

Answers

Divide 9m by 9 which equals 1m then add 3 to both sides and you get m equals 98 which (because you don’t need the 1)

Answer:

m = 98

Step-by-step explanation:

9m/9 - 3 = 95

9m/9 = 95 + 3

9m/9 = 98

cross-multiply

9m = 98 × 9

9m = 882

Divide both sides by 9

m = 98

I need help with this practice problem I need to know if I’m correct.

Answers

Let's draw a picture of our problem:

The trigonometric value function of the angle beta will have the same value of the trigonometric value function of theta. Then, in order to find the cosine of beta, we can use the following right triangle

Therefore,

[tex]\cos \beta=\cos \theta=\frac{-4}{\sqrt[]{61}}[/tex]

or equivalently

[tex]\cos \beta=\frac{-4}{61}\sqrt[]{61}[/tex]

Therefore, the answer is correct.

Hi can you please help meThe cut off part:On the same grid, line k passes through

Answers

line j and k are perpendicular (option B)

Explanation:

J passes through points (8, 2) and (-2, -2)

line K passes through (-4, 3) and (-6, 8)

We need to find the relationship betwen the lines by using the slope from both lines

slope formula is given by:

[tex]m\text{ = }\frac{y_2-y_1}{x_2-x_1}[/tex]

Let's find slope of each line:

[tex]\begin{gathered} \text{for line J: }x_1=8,y_1=2,x_2=-2,y_2\text{ = -}2 \\ \text{slope = m = }\frac{-2-2}{-2-8} \\ \text{slope = }\frac{-4}{-10} \\ \text{slope = 2/5} \\ \\ \text{for line K: }x_1=-4,y_1=3,x_2=-6,y_2\text{ = 8} \\ \text{slope = m = }\frac{8-3}{-6-(-4)} \\ \text{slope = }\frac{5}{-6+4}\text{ = 5/-2} \\ \text{slope = }\frac{\text{-5}}{2} \end{gathered}[/tex]

For two lines to be parallel, their slope will be the same:

Since the slopes are not the same, they are not parallel

For two lines to be perpendicular, the slope of one line will be negative reciprocal of the other line:

slope of one line = 2/5

reciprocal of the line = 5/2

negative reciprocal of the line = -5/2

We can see -5/2 is the slope of the other line.

As aresult, line j and k are perpendicular

Use this diagram to answer the questions
4b. 3
Part A
Which expression represents the area of the rectangle?
B. 6+ (40 - 3)
A6(4-3)
D. 2 x 6 x (40 - 3)
C
6+ (40 - 3) + 6 + (40 - 3)


Part B
Which expression is equivalent to the expression you chose in Part A?
B. 246 - 18
A
240-3
D859
C. 80+ 6

Find the area of the rectangle if = 4 Enter your answer in the box
square units

Answers

We will answer the question given in the picture.

We can see from the question a part of a linear function, and we can see an open circle at the point (4, -2). We can also see that the arrow of the linear function indicates that the function continues infinitely.

To find the domain and the range of the function we need to remember that:

• The domain of a function is, roughly speaking, all of the values for which the function is defined. In general, is represented by all of the values of x for which this function is defined.

,

• The range of a function is, roughly speaking, all the values that the variable y, the dependent variable, takes for each of the values of the independent variable, x.

Therefore, if we check the graph, we have:

The domain of the function

1. The values for x are not defined for x = 4 (we can see a small open circle at the point (4, -2). However, the values for x continue infinitely after that. Therefore, the domain of the part shown is as follows:

[tex]\text{ Domain=}x>4[/tex]

And we can say that the domain of the function is for all of the values greater (not equal to x = 4) to positive infinity. We can write this in interval notation as follows:

[tex]\text{ Domain=}(4,\infty)[/tex]The range of the function

We can check from the graph that the values for y start from y = -2. However, y = -2 is not included since we have a small open circle that indicates that (see above).

Therefore, the range of the function is given by:

[tex]y<-2[/tex]

And we can say that the values of the range are less than y = -2 (not equal), and they are all smaller than y = -2 (for instance, -3, -4, -5.001, -10.222, and so on). The latter values are less than y = -2. We can write this in interval notation as follows:

[tex]\text{ Range=}(-2,-\infty)_[/tex]

Therefore, in summary, we can say that:

1. The inequality to represent the domain of the part shown is x > 4. It means that the domain is those values of the independent variable greater than x = 4 (not equal to 4), and these values extend to positive infinity.

2. The inequality to represent the range of the part shown is y < -2. It means that the range is those values of the dependent variable less than y = -2 (not equal to y = -2), and these values extend to negative infinity.

Are these events independent or dependent? A. Rolling a 6-sided dice twice and recording whether the outcome was even or odd. B. Picking a number from a hat, recording the number and keeping the slip out of the hat, then picking a second number from the same hatA. dependent ; independentB. independent; dependentC. independent ; independentD. dependent ; dependent

Answers

[tex]undefined[/tex]

After completing the fraction division 5 divided by 5/3, Miko used the multiplication shown to check her work. 3x5/3=3/1x5/3=15/3 or 5

Answers

Answer:

Miko found the correct quotient and checked her work using multiplication correctly

Explanation:

When we divided 5 by 5/3, we get:

[tex]5\div\frac{5}{3}=5\times\frac{3}{5}=\frac{5\times3}{5}=\frac{15}{5}=3[/tex]

Therefore, the quotient is 3.

Then, to check the division, we need to multiply the quotient 3 by the divisor 5/3. If we get the dividend 5, the division was correct, so

[tex]3\times\frac{5}{3}=\frac{3}{1}\times\frac{5}{3}=\frac{15}{3}=5[/tex]

Therefore, Miko found the correct quotient and checked her work using multiplication correctly.

A large survey of countries including the USA, China, Russia, France, and others indicated that most people prefer the color blue. In fact, about 24% of the population claim blue as their favorite color. Suppose a random sample of 86 college students were surveyed and 30 of them said that blue is their favorite color. Does this information imply that the color preference of all college students is different (either way) from that of the general population? Use α = 0.05.1. State your null and alternate hypothesis. What is the level of significance? Will you use a left tail, right tail or two-tail test?2. What is the value of the test statistic. 3.lFind the P-value. Sketch the sampling distribution z = to show the area corresponding to the P-value. 4. Based on 1-3, will you reject or fail to reject the null hypothesis? Are the data statistically significant at level α ? State your conclusion in the context of the application.

Answers

We have to perform an hypothesis test of a proportion.

The claim is that the sample has a different proportion than the population.

Then, the null and alternative hypothesis are:

[tex]\begin{gathered} H_0\colon\pi=0.24 \\ H_a\colon\pi\neq0.24 \end{gathered}[/tex]

The significance level is 0.05.

The sample has a size n=86.

The sample proportion is p=0.349.

[tex]p=X/n=30/86=0.349[/tex]

The standard error of the proportion is:

[tex]\begin{gathered} \sigma_p=\sqrt{\dfrac{\pi(1-\pi)}{n}}=\sqrt[]{\dfrac{0.24\cdot0.76}{86}} \\ \sigma_p=\sqrt{0.002121}=0.046 \end{gathered}[/tex]

Then, we can calculate the z-statistic as:

[tex]z=\dfrac{p-\pi-0.5/n}{\sigma_p}=\dfrac{0.349-0.24-0.5/86}{0.046}=\dfrac{0.103}{0.046}=2.241[/tex]

This test is a two-tailed test*, so the P-value for this test is calculated as:

[tex]\text{P-value}=2\cdot P(z>2.241)=0.025[/tex]

* We use a two-tailed test because we are looking for difference above or below the population proportion.

As the P-value (0.025) is smaller than the significance level (0.05), the effect is significant.

The null hypothesis is rejected.

At a significance level of 0.05, there is enough evidence to support the claim that the sample has a different proportion than the population.

Answer:

1) The null and alternative hypothesis are:

[tex]\begin{gathered} H_0\colon\pi=0.24 \\ H_a\colon\pi\neq0.24 \end{gathered}[/tex]

2) The test statistic is z=2.241.

3) The P-value is 0.025. The value in the standard normal distribution is:

4) As the effect is significant (the P-value is less than the significance level), there is evidence to reject the null hypothesis.

The conclusion is that this sample has a proportion that is significantly different from that from the population.

what is represented by 2 in the ordered pair (2,7)

Answers

In (2, 7), 2 is the input

The length of an arc of a circle measures 0.3km. The radius of the circle measures 0.7km. What is the degree measure of the central angle of a circle associated with this arc? Use 3.14 for Π. Round your answer to the nearest tenth.

Answers

SOLUTION:

Step 1:

In this question, we are given the following:

The length of an arc of a circle measures 0.3km.

The radius of the circle measures 0.7km.

What is the degree measure of the central angle of a circle associated with this arc? Use 3.14 for Π.

Round your answer to the nearest tenth.

Step 2:

The details of the solution are as follows:

[tex]\begin{gathered} \text{Length of an arc of a circle = 0. 3 }km \\ \text{Radius of the circle = 0. 7 }km \\ \text{Degr}ee\text{measure of the central angle of a circle = }\theta \\ \pi\text{ = 3. 14} \end{gathered}[/tex][tex]\begin{gathered} \text{Length of Arc , l = }\frac{\theta}{360^0\text{ }}\text{ x 2}\pi r \\ 0.\text{ 3 = }\frac{\theta}{360^0}\text{ x 2 x 3. 14 x 0.7} \end{gathered}[/tex][tex]\begin{gathered} 0.3\text{ = }\frac{\theta\text{ x 4.396}}{360^0} \\ \end{gathered}[/tex]

cross-multiply, we have that:

[tex]\begin{gathered} 360\text{ x 0. 3 = 4.396}\theta \\ \text{Divide both sides by 4.396, we have that:} \end{gathered}[/tex][tex]\begin{gathered} \theta\text{ = }\frac{360\text{ X 0. 3}}{4.396} \\ \end{gathered}[/tex][tex]\begin{gathered} \theta=\text{ }\frac{108}{4.396} \\ \end{gathered}[/tex][tex]\begin{gathered} \theta\text{ = 24.5677889} \\ \theta\approx24.6^{0\text{ }}(\text{ to the nearest tenth)} \end{gathered}[/tex]

Seventh grade > Y.12 Area of compound figures with triangles MRGWhat is the area of this figure?3 mm4 mm8 mm2 mm5 mm6 mm3 mmWrite your answer using decimals, if necessary.square millimeters6 mm

Answers

Answer:

Area of the figure = 72mm²

Explanation:

Given:

An irregular figure

To find:

the area of the figure

To determine the area, we will divide the figure into shapes with known areas

We have 1 triangle, 1 rectangle, and 1 square. We will find the area of each of the shapes

Area of triangle = 1/2 × base × height

height = 3mm

base = 4 + 3 + 1 = 8mm

[tex]\begin{gathered} Area\text{ of the triangle = }\frac{1}{2}\times8\times3 \\ \\ Area\text{ of the traingle = 12 mm}^2 \end{gathered}[/tex]

Area of rectangle = length × width

length = 8mm, width = 3mm

[tex]\begin{gathered} Area\text{ of the rectangle = 8 }\times\text{ 3} \\ \\ Area\text{ of the rectangle = 24 mm}^2 \end{gathered}[/tex]

Area of square = length²

length = 6 mm

[tex]\begin{gathered} Area\text{ of the square = 6}^2 \\ \\ Area\text{ of the square = 36 mm}^2 \end{gathered}[/tex]

Area of the figure = Area of triangle + Area of the rectangle + Area of the square

[tex]\begin{gathered} Area\text{ of the figure = 12 + 24 + 36} \\ \\ Area\text{ of the figure = 72 mm}^2 \end{gathered}[/tex]

2 3/5 •3= ?A 13/15B 1 2/13C 6 3/5D 7 4/5

Answers

Given:

[tex]2\frac{3}{5}\times3=?[/tex]

First of all,we multiply 5 and 2 i.e.

[tex]\begin{gathered} 5\times2=10 \\ \end{gathered}[/tex]

Then,add 3 to this value.

It becomes 10+3=13

Hence,

[tex]2\frac{3}{5}\times3=\frac{13}{5}\times3[/tex][tex]=\frac{39}{5}[/tex]

Now, we convert the obtained value in mixed fraction.

First we divide 39 by 5 and then write the remainder in numerator and dividend in the side as shown.

[tex]\frac{39}{5}=7\frac{4}{5}[/tex]

Here, when we divide we get 6 as divident and 4 as remainder, so we express it like this.

Hence, option (4) is correct.

When a tow truck is called, the cost of the service is $150 plus $5 per mile that the car must be towed.
Write and graph a linear equation to represent the total cost of the towing service, which is dependent on the number of miles the car is towed.
Find and interpret the slope and y-intercept of the linear equation

Answers

The linear equation to represent the total cost of the towing service is;

C = 150 + 5x.

What is defined as the term linear equation?A linear equation is one in which the variable's highest power is always 1. A one-degree equation is another name for it. A linear equation inside one variable has the standard form Ax + B = 0.

For the given question,

The fixed cost of towing service = $150.

The variable cost per mile = $5.

Let the number of miles be 'x'.

The total cost be 'C'.

Thus, the equation becomes;

C = 150 + 5x.

C = 5x + 150 (linear equation to represent the total cost of the towing service)

Find the slope and  y-intercept of the linear equation;

Comparing the equation with the slope intercept form of line;

y = mx + c

slope m = 5

y-intercept c = 150.

Thus, the  linear equation to represent the total cost of the towing service is; C = 150 + 5x.

To know more about the linear equation, here

https://brainly.com/question/2030026

#SPJ1

Which statements describe one of the transformations performed on f(x) = x²to create g(x) = 3(x+5)² -2? Choose all that apply.A. A vertical stretch with a scale factor of 3B. A translation of 2 units to the leftC. A translation of 5 units to the left□ D. A vertical stretch with a scale factor of eP

Answers

The transformations performed on the function f(x) to create g(x) include : a vertical stretch with a scale factor of 3 and a translation of 5 units to the left.

We are given a function f(x). The function f(x) is defined as x². We also have another function, g(x). The function g(x) is defined as g(x) = 3(x + 5)² - 2. The function g(x) is formed by performing several transformations on the function f(x). The first transformation is translating the function to the left by 5 units. The next transformation is stretching the function vertically by a scale factor of 3. The last transformation is translating the function downward by 2 units.

To learn more about transformations, visit :

https://brainly.com/question/11709244

#SPJ9

fifty four percent of the items in a refrigerator are dairy products what percent of the items are non dairy products

Answers

Given :

54% of the items are dairy products.

Find the absolute maximum and absolute minimum values of f on the given interval.

f(x) = xe^(−x^2/98), [−3, 14]

absolute minimum value?
absolute maximum value?

Answers

Absolute minimum value and maximum value at f(-3) = -2.7 and

f(14) = 12.14 respectively.

Define function.

An association between a number of inputs and outputs is called a function. A function is, to put it simply, an association of inputs where each input is connected to exactly one output. For each function, there is a corresponding range, codomain, and domain.

Given function is -

f(x) = x*e^(−x^2/98)

By differentiating the function, we will get

f'(x) = (1)([tex]e^{-x^{2} /98}[/tex])+ x([tex]\frac{-2x}{98}[/tex] *    [tex]e^{-x^{2} /98}[/tex])

f'(x) = ([tex]e^{-x^{2} /98}[/tex] ) - ([tex]\frac{x^{2} }{49}[/tex] * [tex]e^{-x^{2} /98}[/tex])

f'(x) = ([tex]e^{-x^{2} /98}[/tex]) (1 - [tex]\frac{x^{2} }{49}[/tex])

To calculate the maximum and minimum value, (1 - [tex]\frac{x^{2} }{49}[/tex]) must be zero or              ([tex]e^{-x^{2} /98}[/tex]) must be zero.

=> (1 - [tex]\frac{x^{2} }{49}[/tex]) =0

=> [tex]\frac{x^{2} }{49}[/tex] = 1

=> [tex]x^{2}[/tex] = 49

=> x = 7 or x= -7

However, -7 is not within our given interval and does not need to be tested. Therefore, put the x = -3,7,14 in given function.

f(-3) = -3 [tex]e^{-9/98}[/tex] = -2.7

f(7) = 7 [tex]e^{-1/2}[/tex] = 4.24

f(14) = 14 [tex]e^{-1/7}[/tex] = 12.14

Absolute minimum value at f(-3) = -2.7 and

Absolute maximum value at f(14) = 12.14

To know more about function, visit:

https://brainly.com/question/14364803

#SPJ1

Other Questions
GeometryQuestion 5For what values of the variables must ABCD be a parallelogram? In "Song for the Turtles in the Gulf" how does Linda Hogan's diction develop the theme? Solve forj: -16 = j/2 + -19 A person walks 15.0 m in 5.00 s and thenwalks 12.0 m in 10.00 s. What is theaverage speed of the person? how you can contribute and help to solve the problem on water crisis at present and in the future What is the mass in grams of 5.024x23 platinum atoms? Solve the following system of equations graphically on the set of axes below.y = -1/3x - 4 y = 2/3x + 2 Read the dictionary definition for the word awe.awe \ o \ noun: a strong feeling of fear, respect, or wonderBased on this definition, what is most likely a definition of awesome?Ocausing feelings of indifferenceO lacking enough courageO inspiring great admirationcausing feelings of weakness juan has fewer than ten $10 bills. he has more tahn three $1 bills. he has twice as many tens as ones. how much money does he have? Find the quotient and remainder using long division.x4 5x3 + x 4 / x2 7x + 1 Simply the expression 11s(4) Use the distributive property to write an equivalent expression. If you get stuck, consider drawing a diagram p( 4p + 9) complementary angles. If m/A= (5x - 11) andZA and ZB arem/B = (6x + 2), then find the measure of ZA.PLEASE ANSWER ASAP !!!! Find the value of X and each arc measurex =mGK=mHJ = mHGJ =mGKJ= describe one mitigation strategy that humans can emply to rduce the native impact of human population growth on forests In the figure, BC||DE Angles_____1.CAF and EFA2.GAC and DFE3.CAF and EFH4.GAB and EFAare congruent______1.By the linear pair Theorem.2. because they are corresponding angles of parallel lines cut by transversal.3. by the vertical angles theorem.4. by the transitive property of congruence.GAC - CAFE because they are corresponding angles of parallel lines cut by a transversal.LAFE - HFD by the Vertical Angles Theorem.GAC - HFD by the_____1. Addition 2. Subtraction3. Substitution4. TransitiveProperty of Congruence. If the son with ald in the third generation survives to have children with a woman without any history of ald in her family, what is the chance that a newborn son of this couple will not develop ald?. Type the correct answer in each box. Use numerals instead of words.Sabrina is researching the growth of a population of horses on a ranch. She models the population of horses using the function below, where n is the number of years after she begins the research and b is an unknown base. - 1/3 x = 3/4solve for x 1. 1/42. - 1/43. 2 1/44. -2 1/4 The following data values represent a population. What is the variance of thepopulation? = 11. Use the information in the table to help you.X8101214(x-)9119OA. 10B. 5O C. 11OD. 20